Hardest question on this channel solved some easiest of concepts

Поделиться
HTML-код
  • Опубликовано: 23 окт 2024

Комментарии • 91

  • @jeesimplified-subject
    @jeesimplified-subject  4 месяца назад +11

    The answer is n = 337, for m = 56.
    Also, send us the hardest question you have ever tried here 👇🏻
    forms.gle/HZxgUAKdWV1Pywgb8

    • @steveop1821
      @steveop1821 4 месяца назад

      *n=337

    • @akaalkiratsingh
      @akaalkiratsingh 4 месяца назад

      Bhaiya how are you able to say :
      n2 is integral
      U said n2 is(n1)/3

    • @YadavRitesh-wg7ip
      @YadavRitesh-wg7ip 4 месяца назад +1

      I have sended a fantastic problem which is of moderate to difficult level but is extremely tricky

    • @YadavRitesh-wg7ip
      @YadavRitesh-wg7ip 4 месяца назад +1

      Pls make the next video on the problem that I have sended😊❣️

  • @mdsalsabeel6046
    @mdsalsabeel6046 4 месяца назад +30

    That's exactly the solution i posted in group.... It's still there you'll can check
    Mene ise bina pells k hi kia tha i used number theory

    • @jeesimplified-subject
      @jeesimplified-subject  4 месяца назад +25

      Congrats, If you cracked it 😄

    • @mdsalsabeel6046
      @mdsalsabeel6046 4 месяца назад +4

      @air02IITB haa me whi hu

    • @mdsalsabeel6046
      @mdsalsabeel6046 4 месяца назад

      @air02IITB nhi bhai me sirf 2025 aspirant hu olympiad wagera k bare me kch pta nhi h mujhe filhal btw thank you

    • @anaygoyal1657
      @anaygoyal1657 4 месяца назад +1

      Can you tell your solution?

    • @mdsalsabeel6046
      @mdsalsabeel6046 4 месяца назад

      @@anaygoyal1657 it's of one nd half a page, how do i share that to u?

  • @satyapalsingh4429
    @satyapalsingh4429 4 месяца назад +10

    (n+1)(2n+1)=6k^2. Let n+1=2p^2 and 2n+1=3q^2.So k=pq.Eliminating n ,we get x^2-3q^2=1 which is a Pell’s equation (Here x= 2p). x=26 and q=15 are the solutions of x^2-3q^2=1 .So n=337 and k=195. Second solution is n=2465845537 .Here p=35113 and q=40545 and k=pq =1423656585.

    • @FocusonSolution780
      @FocusonSolution780 4 месяца назад

      Bro here you have assumed that 2n + 1 is a multiple of 3. But what if it is not? It is possible that (n + 1) itself contains both 2 and 3 in its prime factorisation

    • @santanuganguly5642
      @santanuganguly5642 4 месяца назад

      @@FocusonSolution780 fir 2n+1. 4k-1 ki form mai ayega joki kabhi bhi perfect square nhi hai

  • @fgblaze7165
    @fgblaze7165 4 месяца назад +4

    7:51 Bhaiya binomial theroem me remainder wale question karte hai to hum use multiple±1 ke term me break karte hai aur power n raise karte hai yaha n 1 ho gayega aur agar n2 'd' se divide ho rha hai to 12 n2 bhi hoga to use hume multiple ±1 ke form me tod liya aur to remainder 1/d agya to humne prove kar diya dono co prime hai

  • @aniruddhxie2k215
    @aniruddhxie2k215 4 месяца назад +9

    16:50 Isme 196 bhi ek possibility hai for 3m+1

  • @CodeWithBhaskar25
    @CodeWithBhaskar25 4 месяца назад +7

    I solved it using python code
    for i in range(1, 1000): # start from 1 to avoid division by zero
    s = (((i * (i + 1) * ((2 * i) + 1)) / 6) / i)**(1/2)
    if s.is_integer():
    print(i, s)
    # else:
    # print('no')

    • @vanshagrawal6847
      @vanshagrawal6847 4 месяца назад +6

      😂

    • @skies_06
      @skies_06 4 месяца назад +8

      dekho isko 🤦‍♂🤣

    • @hanshalghag2394
      @hanshalghag2394 4 месяца назад +6

      Bhai exam me command prompt kholega💀

    • @Barnil_JN
      @Barnil_JN 4 месяца назад +5

      To the people demotivating him in the replies...
      I mean just see, bro just did something very cool. Bro naturally talented in coding and isn't it a good thing that a person has some hobbies other than runnin' in the ratrace?

    • @Barnil_JN
      @Barnil_JN 4 месяца назад

      Btw keep up bro!

  • @anvaysrivastava1114
    @anvaysrivastava1114 4 месяца назад +1

    GCD(k,12k-1)=GCD(k,12k-1-11k)=GCD(k,k-1)=1 because GCD of two consecutive numbers is always 1

  • @findmebro
    @findmebro 4 месяца назад +18

    please don't get disappointed by less views and stop posting please. love you

    • @pulkitgupta5550
      @pulkitgupta5550 4 месяца назад +1

      Stop posting??? You good buddy?

    • @steadytuna8753
      @steadytuna8753 4 месяца назад

      @@pulkitgupta5550 he w anted to say " please dont stop posting because of less views"

  • @amaldev5970
    @amaldev5970 4 месяца назад +6

    17:11 196 is also a possible candidate for 3m+1 right?

  • @drdripransom342
    @drdripransom342 4 месяца назад +3

    Yeh sab maine socha tha, like literally co primes hoyenge and all that, lekin aage kaise badhe woh nhi ho paya rha tha, waise without substitutions bhi hojata I guess

  • @Astro-r5z
    @Astro-r5z 4 месяца назад +16

    And i was thinking ki mujhe hi video nahi mil rahi😂 post hi aaj kari hai😵‍💫

  • @archanashrivastava7281
    @archanashrivastava7281 4 месяца назад +58

    Am I the only one jisko yeh solution dekhke darr lag raha hai 💀

  • @arenacoder
    @arenacoder 4 месяца назад +3

    Raw math indeed

  • @AnjaliYadav-xg2no
    @AnjaliYadav-xg2no 4 месяца назад +1

    14:55
    maine tg pr apna solution bheja tha mai bhi usme iss conclusion tk aa gya tha , just after this step we can convert it into pell's equation for a more synthetic solution. Anyway good Question

  • @adi7
    @adi7 6 дней назад

    end mein values daal ke solve kr dia... proper solution waali feel nahi aai... uske aage bhi thodi number theory laga ke proper solve kr skte the

  • @amaldev5970
    @amaldev5970 4 месяца назад +1

    Bhaiyya, this is similar to a BMO 2 question 1 in the 90s i guess (probably 1992-93). I am very sure that you would have taken this question from an olympiad based resource!. Happy Problem solving :)

  • @vairagyapathak5304
    @vairagyapathak5304 23 дня назад

    8:53 What if n2 is 1?

  • @dipanshuchandel5903
    @dipanshuchandel5903 4 месяца назад

    Interesting question bhai sare gyan chakshu khul gaye

  • @MitanshilovecheesecakeMakwana
    @MitanshilovecheesecakeMakwana 2 месяца назад

    maza baandh diya bro

  • @shivanshnigam4015
    @shivanshnigam4015 4 месяца назад +1

    bhaiya yeh sawaal wahi solve kar payega basic number theory use karke jo baith ke khelta ho numbers se paper pe, without using any hardcore number theory

  • @YadavRitesh-wg7ip
    @YadavRitesh-wg7ip 4 месяца назад +2

    I also posted right answer in group

  • @Soul-cu8zn
    @Soul-cu8zn 4 месяца назад +2

    Bhai ye intro problem level h oly k liye

  • @wdivyankop
    @wdivyankop 4 месяца назад

    bhaiya maine root me 6 ko 2 x 3 me baant liya aur usse hit and trial se 337 hi ans aaya under 5 min

  • @bhoju_
    @bhoju_ 4 месяца назад

    Bring more such problems

  • @bhoju_
    @bhoju_ 4 месяца назад +1

    Getting 40-50 marks in maths jee advance 2024, jee mains 98%ile in maths

    • @dictetord12
      @dictetord12 4 месяца назад

      17 marks in jee mains April attempt
      But 40 marks in adv 2024

    • @bhoju_
      @bhoju_ 4 месяца назад

      @@dictetord12 nice

    • @Sudhanshu_Kumar11
      @Sudhanshu_Kumar11 4 месяца назад

      ​@@dictetord12 Kitni rank aa jayegi bro

  • @18football18
    @18football18 4 месяца назад +5

    16:33 196 bhi hoga 3m+1 form ka😢😢😢😢😢

    • @bhoju_
      @bhoju_ 4 месяца назад

      Aur multiplication me satisfy krega kya voh?

  • @shivuveerkumar
    @shivuveerkumar 4 месяца назад +1

    Bro which white board software you are using

  • @Mr.X.
    @Mr.X. 4 месяца назад

    Bhai ni smjh aaya 15:39
    Iske liye kon kon se chapters clear hone chiye
    I'm in 11th

  • @ancientancedotes
    @ancientancedotes 4 месяца назад

    this is a British Mathematics OLympiad qs

  • @chandadevibairwa7287
    @chandadevibairwa7287 2 месяца назад

    My approach
    (N+1)(2N+1) =6K^2
    Therefore (N+1)(2N+1) sould be divisible by 6
    Let N = 6a+r where r belons to {-2,-1,0,1,2,3}
    Satisfying values we get r=1 or r=-1
    Therefore N=6a+1 or 6a-1
    Taking first case
    Then (6a+2)(12a+3)=6k^2
    (3a+1)(4a+1) = K^2
    Ab same process but is equation ko lane me itna dimag lagane ki jarurat nhi h iske bad vhi dono co prime h to dono ko perfect sqaure hona padega simultaneously for which we get a=56
    Therefore N=6×56+1 =337.

  • @shivanshnigam4015
    @shivanshnigam4015 4 месяца назад +1

    196 bhi 3m+1 ke type ka hai

  • @chandranisahanone
    @chandranisahanone 4 месяца назад

    Nice question!!❤

  • @syed3344
    @syed3344 4 месяца назад

    We aren't taught number theory.

  • @maulikshah28
    @maulikshah28 4 месяца назад

    Amazing

  • @__onii__sama__
    @__onii__sama__ 4 месяца назад

    Maza aagaya kasam se

  • @Gyan-fx9zx
    @Gyan-fx9zx 4 месяца назад

    Pells equation se do minute me solve ho jayega

  • @cofanavay2235
    @cofanavay2235 Месяц назад

    i am into iitgbjust for no reason i am watching this

  • @ombhukal8358
    @ombhukal8358 4 месяца назад

    Bhai ye to number theory ka question hai

  • @seulideymallik7309
    @seulideymallik7309 4 месяца назад

    Me khud ek solution kara hoo bo kya share karuun ?

  • @sayanNITdgp2025
    @sayanNITdgp2025 4 месяца назад

  • @saranshchakrawarti9040
    @saranshchakrawarti9040 4 месяца назад +1

    😵😵😵kuch samajh nahi aya

  • @adityarajkanth
    @adityarajkanth 4 месяца назад +3

    Honestly idk abt the question but the solution was really baseless, baseless in the sense ki it won't help any drop adding to ur iq or in ur jee it was just a typical Olympiad question with a non so formal solution, for me it was a waste of time, im saying all this because of why u were so happy with ur soln

    • @jeesimplified-subject
      @jeesimplified-subject  4 месяца назад +3

      Indeed, thoda out of the box chalagaya par the fun part is koi typical identity use ni hui.
      But i don’t agree with IQ vala point and raw creativity it holds is bound to increase ones depth.

    • @adityarajkanth
      @adityarajkanth 4 месяца назад

      @@jeesimplified-subject iq wala point can be added to the one jo ye manipulation hi krta hai sirf jisko aur new cheeze mili but as a jee aspirant it doesn't help right

    • @divyanshgarg5434
      @divyanshgarg5434 4 месяца назад +1

      ​@@adityarajkanth why do you think so one dimensionally? Do you need to do each and every thing that perfectly "aligns with the syallabus" and "helps" in JEE? Can't there be room for enjoyment of mathematics?

    • @adityarajkanth
      @adityarajkanth 4 месяца назад

      @@divyanshgarg5434 bhai phir tu kuch zyada hi free h

    • @divyanshgarg5434
      @divyanshgarg5434 4 месяца назад

      @@adityarajkanth You can say that, as I had started my prep in 9th

  • @khalidjamonday_2670
    @khalidjamonday_2670 2 месяца назад +1

    Bhai tujhe samjhana ni ata bilkul smjh ni aya solution dekh k mtt smjhaya kr

  • @__onii__sama__
    @__onii__sama__ 4 месяца назад +3

    Heh... NEET maths level question 🤡

  • @CodeWithBhaskar25
    @CodeWithBhaskar25 4 месяца назад

    by the way one solution is 65521 also